I need to know the system of equation in the photo

I Need To Know The System Of Equation In The Photo

Answers

Answer 1

Solution:

The graph has a solution (4,-1);

That is, the system of equation must satisfy x=4 as y=-1.

LINE 1 has its y-intercept;

[tex]\begin{gathered} (0,1) \\ \end{gathered}[/tex]

LINE 2 has its y-intercept as;

[tex](0,-5)[/tex][tex]\begin{gathered} x+2y=2 \\ 2y=-x+2 \\ y=-\frac{1}{2}x+\frac{2}{2} \\ y=-\frac{1}{2}x+1 \\ \\ x-y=5 \\ y=x-5 \end{gathered}[/tex]

Thus, the system of equation that satisfy the graph is;

[tex]\begin{gathered} x-y=5 \\ x+2y=2 \end{gathered}[/tex]

I Need To Know The System Of Equation In The Photo

Related Questions

ActiveApplying the Triangle Inequality Theoremin triangle ABC, AB measures 25 cm and AC measures 35 cm.The inequalitycentimeters.

Answers

Using the Triangle inequality:

[tex]zso:

[tex]undefined[/tex]

There are 14 girls and 12 boys in a class. What is the ratio of gris to students in simplest form

Answers

Number of girls = 14

Number of boys = 12

Number of students = 14 + 12 =26

Ratio of girls to students = 14/26 = 7 : 13

4^2 * 4^3 simplified

Answers

SOLUTION

Given the question in the image, the following are the solution steps to answer the question.

STEP 1: Write the given expression

[tex]4^2\ast4^3[/tex]

STEP 2: Simplify the expression using the law of indices

[tex]\begin{gathered} \mathrm{Apply\:exponent\:rule}:\quad \:a^b\cdot \:a^c=a^{b+c} \\ 4^2\cdot \:4^3=4^{2+3} \\ =4^{\left\{2+3\right\}} \\ =4^5=1024 \end{gathered}[/tex]

Hence, the evaluation gives:

[tex]1024[/tex]

if frita goes to the mall, then alice will go to the mall

Answers

Given

The statements,

If Frita goes to the mall, then Alice will go to the mall.

If Wally goes to the mall, then Frita will go to the mall.

To find: The conclusion using the law of Syllogism.

Explanation:

It is given that,

If Frita goes to the mall, then Alice will go to the mall.

If Wally goes to the mall, then Frita will go to the mall.

That implies,

If Wally goes to the mall, then Frita will go to the mall.

If Frita goes to the mall, then Alice will go to the mall.

Here, consider the statement Wally goes to the mall as p, the statement Frita will go to the mall as q, and the statement Alice will go to the mall as r.

Therefore,

[tex]Conclusion:\text{ }If\text{ }Wally\text{ }goes\text{ }to\text{ }the\text{ }mall,\text{ }then\text{ }Alice\text{ }will\text{ }go\text{ }to\text{ }the\text{ }mall.[/tex]

Hence, the answer is option C).

the five number summary for a set of data is given in the picture.what is the interquartile range of the set of data?

Answers

ANSWER

[tex]IQR=13[/tex]

EXPLANATION

The interquartile range can be found by finding the difference between the first quartile, Q1, and the third quartile, Q3.

That is:

[tex]IQR=Q3-Q1[/tex]

Therefore, the interquartile range is:

[tex]\begin{gathered} IQR=81-68 \\ IQR=13 \end{gathered}[/tex]

Which of the following graphs could be a representation of a geometric sequence?Check all that apply.A.B.C.D.

Answers

SOLUTION:

We want to find the graph corresponding to a geometric sequence.

The equation of a geometric sequence is;

[tex]a_n=a_1(r)^{n-1}[/tex]

This is clearly an exponential function with a starting value a.

The correct graphs are OPTION B and OPTION D

Choose the correct answer. 1. Shari made a net of a box to find how much wrapping paper she will need to . wrap the box 8 in. 5 in.

Answers

The total area needed to be covered is 158 square inch.

It is calculated by adding all the area

[tex]2\ast(8\ast5)+\text{ 2}\ast(5\ast3)+2\ast(8\ast3)=158in^2\text{ }[/tex]

write an equation in slope intercept form of the line that passes through the given point and is perpendicular to the graph of the given equations(0,0); y = -6x+3y =

Answers

To find the equation of the line we need a point and the slope. We have the point but we need to find the slope, to do this we need to remember that two lines are perpendicular if and only if their slopes fullfils:

[tex]m_1m_2=-1[/tex]

Now, the slope of the line given is -6, this comes from the fact that the line is written in the form y=mx+b, hence comparing both equation we conclude that.

Pluggin this value into the condition above we have:

[tex]\begin{gathered} -6m_1=-1 \\ m_1=\frac{-1}{-6} \\ m_1=\frac{1}{6} \end{gathered}[/tex]

Therefore the slope of the line we are looking for is 1/6. The equation of a line is given as:

[tex]y-y_1=m(x-x_1)[/tex]

Plugging the values of the slope and the point we have:

[tex]\begin{gathered} y-0=\frac{1}{6}(x-0) \\ y=\frac{1}{6}x \end{gathered}[/tex]

Therefore the equation we are looking for is:

[tex]y=\frac{1}{6}x[/tex]

List the sides of FGH in order from least to greatest if m

Answers

We have a triangle FGH

The three angles of the triangle FGH are given as

[tex]\begin{gathered} m\angle F=4x+7 \\ m\angle G=5x-31 \\ m\angle H=7x-52 \end{gathered}[/tex]

Recall that the sum of all three angles of a triangle must be equal to 180°

[tex]\begin{gathered} m\angle F+m\angle G+m\angle H=180\degree \\ 4x+7+5x-31+7x-52=180 \\ 16x-76=180 \\ 16x=180+76 \\ 16x=256 \\ x=\frac{256}{16} \\ x=16 \end{gathered}[/tex]

Now, we can calculate the exact measure of the angles

[tex]\begin{gathered} m\angle F=4x+7=4(16)+7=64+7=71\degree \\ m\angle G=5x-31=5(16)-31=80-31=49\degree \\ m\angle H=7x-52=7(16)-52=112-52=60\degree \end{gathered}[/tex]

Let us draw the triangle FGH

Recall that the side opposite the least angle is the least side and vice versa.

The means that the side opposite the angle G is the least side (FH)

Then the side opposite the angle H is the greater side (FG)

Finally, the side opposite the angle F is the greatest side (GH)

Therefore, the sides of the triangle FGH in order from least to greatest is

FH, FG, GH

If Karen has 6 cups of oatmeal and she divides it into 1/3 cup servings, how many servings of oatmeal will she have?

Answers

Problem

If Karen has 6 cups of oatmeal and she divides it into 3/4 cup servings, how many servings of oatmeal will she have? ​

Solution

For this case the operation that we need to do is:

[tex]\frac{6\text{cups}}{\frac{3}{4}\frac{\text{cups}}{\text{serving}}}=\frac{6\cdot4}{3}==\frac{24}{3}=8\text{servings}[/tex]

And for this case the final answer would be 8 servings

In your Grandpa Will's recipe for a marinade, each serving uses 3.5 tablespoons of ketchup and 7 tablespoons of vinegar. If 31.5 tablespoons of ketchup will be used for a larger batch of marinade, how much vinegar is needed? tablespoons of vinegar are needed. Submit answer

Answers

The rate of ketchup to vinegar should be preserved. Let V be the volume of vinegar that will be used for the larger batch of marinade. Since the recipe uses 7 tablespoons of vinegar for each 3.5 tablespoons of ketchup, then:

[tex]\frac{V}{31.5}=\frac{7}{3.5}[/tex]

Then, the volume of vinegar for the larger batch of marinade can be calculated as:

[tex]\begin{gathered} V=\frac{7}{3.5}\times31.5 \\ =63 \end{gathered}[/tex]

Therefore, 63 tablespoons of vinegar are needed.

For each problem below find the missing factor by computing the inverse operation

Answers

Given:

There are given that the fraction:

[tex]4\frac{1}{2}-\text{ \lbrack \rbrack}=2\frac{7}{8}[/tex]

Explanation:

Suppose missing information is x

Then,

Ater that we need to find the value of x

So,

[tex]4\frac{1}{2}-x=2\frac{7}{8}[/tex]

Then,

[tex]\begin{gathered} 4\frac{1}{2}-x=2\frac{7}{8} \\ \frac{9}{2}-x=\frac{23}{8} \\ \frac{9}{2}-x-\frac{9}{2}=\frac{23}{8}-\frac{9}{2} \\ -x=\frac{23}{8}-\frac{9}{2} \end{gathered}[/tex]

Now,

[tex]\begin{gathered} -x=\frac{23}{8}-\frac{9}{2} \\ -x=\frac{23-36}{8} \\ -x=\frac{-13}{8} \\ x=\frac{13}{8} \\ x=1\frac{5}{8} \end{gathered}[/tex]

Final answer:

Hence, the missing factor is shown below:

[tex]x=1\frac{5}{8}[/tex]

Select the correct answer.What is the image of this figure after this sequence of dilations?1. dilation by a factor of -1 centered at the origin2. dilation by a factor of 2 centered at (-1,1)

Answers

The coordinates of the original figure are:

(-2,1)

(3,1)

(1,3)

(-2,3)

A dilation by a negative scale factor produces an image on the other side of the center of enlargement.

As the first dilation is by a factor of -1 centered at the origin, the length of the sides doesn't change, but the new coordinates will be:

[tex](x,y)\to(kx,ky)[/tex]

Apply this to the given coordinates:

[tex]\begin{gathered} (-2,1)\to(-1\cdot-2,-1\cdot1)\to(2,-1) \\ (3,1)\to(-1\cdot3,-1\cdot1)\to(-3,-1) \\ (1,3)\to(-1\cdot1,-1\cdot3)\to(-1,-3) \\ (-2,3)\to(-1\cdot-2,-1\cdot3)\to(2,-3) \end{gathered}[/tex]

The image after the first dilation looks like this:

Now, the second dilation is by a scale factor of 2, centered at (-1,1).

As it is not centered in the origin, we can use the following formula:

[tex](x,y)\to(k(x-a)+a,k(y-b)+b)[/tex]

Where k is the scale factor and (a,b) are the coordinates of the center of dilation.

By applying this formula to the actual coordinates we obtain:

[tex]\begin{gathered} (2,-1)\to(2(2-(-1))+(-1),2(-1-1)+1)\to(5,-3) \\ (-3,-1)\to(2(-3-(-1))+(-1),2(-1-1)+1)\to(-5,-3) \\ (-1,-3)\to(2(-1-(-1))+(-1),2(-3-1)+1)\to(-1,-7) \\ (2,-3)\to(2(2-(-1))+(-1),2(-3-1)+1)\to(5,-7) \end{gathered}[/tex]

If we place these coordinates in the coordinate plane we obtain:

The answer is option B.

Answer:

B .

Step-by-step explanation:

Got the answer right.

[tex]2x ^{2} - 6x + 10 = 0[/tex]solve by completing the square

Answers

We know that we can use the quadratic equation

Using this we have

[tex]\begin{gathered} x=\frac{-(-6)\pm\sqrt[]{(-6)^2-4\cdot2\cdot10}}{2\cdot2}=\frac{6\pm\sqrt[]{36-80}}{4} \\ =\frac{6\pm\sqrt[]{-44}}{4}=\frac{6\pm\sqrt[]{4\cdot-11}}{4}=\frac{6\pm2\cdot\sqrt[]{-11}}{4} \\ =2\cdot(\frac{3\pm\sqrt[]{-11}}{4})=\frac{3\pm\sqrt[]{-11}}{2}=\frac{3\pm\sqrt[]{11}i}{2} \\ =\frac{3}{2}\pm\frac{\sqrt[]{11}}{2}i \end{gathered}[/tex]

So the answer is B)

(calc) which graph shows a function and its inverse ?

Answers

Answer: We have to pick a graph that represents a function and its inverse function, or:

[tex]\begin{gathered} f(x)\rightarrow\text{ and }\rightarrow f^{-1}(x) \\ \end{gathered}[/tex]

The inverse function switches the x and y variables, therefore the axes with it, the final result is two functions that are symmetrical about y = x line.

Example:

[tex]\begin{gathered} f(x)=\sqrt{x}\rightarrow\text{ Function} \\ \\ f^{-1}(x)=x^2\rightarrow\text{ Inverse Function} \end{gathered}[/tex]

Graph:

Therefore the graph out of the options which has these properties is:

[tex]\text{ Graph\lparen C\rparen}[/tex]

The answer, therefore, is Graph(C).

Please help on average rate of change!

Answers

The average rate of change on the interval [-1, 2] is 1/3.

How to get the average rate of change?

For any function f(x), we define the average rate of change on an interval [a, b] as:

r = ( f(b) - f(a))/(b - a)

In this case, the function is graphed, and the interval is [-1, 2]

On the graph we can see that:

f(-1) = -3

and

f(2) = -2

Replacing these we will get:

r = ( f(2) - f(-1))/(2 - (-1))

r = (-2 + 3)/(3) = 1/3

The average rate of change is 1/3.

Learn more about average rates of change:

https://brainly.com/question/8728504

#SPJ1

A) What does the point (1,8) represent in the context of the situation? B) Is the amount of money proportional to the number of hours worked? C) Write an equation that represents this situation? D) What will be Amber’s Wages after 6 hours worked?

Answers

Answer:

A) Amber's wages for 1 hour is $8.

B) Yes

C) y = 8x

D)

Explanation:

A) Looking at the graph, we can deduce that the point (1, 8) shows how much Amber makes in one hour. So in 1 hour, Amber makes $8.

B)To determine whether the amount of money is proportional to the number of hours worked, we have to look at the graph and see if it starts from the origin (0, 0), if it does then we can conclude that they are proportional.

Since the graph starts from the origin (0, 0), then the amount of money is proportional to the number of hours worked.

C) The slope-intercept equation of a line is given as;

[tex]y=mx+b[/tex]

where m = slope of the line

b = y-intercept of the line

So let's go ahead and determine the slope of the line at points (1, 8) and (2, 16) using the below formula;

[tex]m=\frac{y_2-y_1_{}_{}_{}}{x_2-x_1_{}}=\frac{16-8}{2-1}=\frac{8}{1}=8[/tex]

Since the line starts from the origin, therefore the y-intercept, b, is zero.

Since m = 8 and b = 0, the equation can then be written as;

[tex]\begin{gathered} y=8x+0 \\ y=8x \end{gathered}[/tex]

D

The constant of variation for a function is 2. Which of the following graphs best represents this situation

Answers

The required graph shows the constant of variation for a function is 2 is A and B. Option A and B is correct.

Given that,
To determine the graphs which show the constant of variation for a function is 2.

What is proportionality?

proportionality is defined as between two or more sets of values, and how these values are related to each other in the sense are they directly proportional or inversely proportional to each other.

here,
in the graphs only graph, A and B show the given condition of the constant of variation for a function is 2. Because in both graphs shows that y = 2x  and 2y = x.

Thus, the required graph shows the constant of variation for a function is 2 is A and B. Option A and B is correct.

Learn more about proportionality here: https://brainly.com/question/22620356

#SPJ1



simple interest interest: $50principal:$350rate: 6.5time:x

Answers

Given:

Interest (SI) = 50

Principal (P) = 350

rate (R) = 6.5. (Here, rate is 6.5 %)

To find time,

[tex]\begin{gathered} S\mathrm{}I\mathrm{}=P\times R\times T \\ 50=350\times\frac{6.5}{100}\times x \\ x=\frac{50}{22.75} \\ x=2.197 \\ x\approx2\text{ years ( approximated) } \end{gathered}[/tex]

Answer: x = 2 years.

Suppose that y varies directly as the square root of x, and that y = 25 when x = 289. What is y when x= 134? Round your answer to two decimal places if necessary.

Answers

Answer

When x = 134, y = 17.02

Explanation

We are told that y varies directly as the square root of x.

y ∝ √x

Introducing the constant of proportionality, k

y = k√x

We are then told that

when y = 25, x = 289, with this, we can solve for k

y = k√x

25 = k × √289

25 = k × 17

25 = 17k

17k = 25

Divide both sides by 17

(17k/17) = (25/17)

k = (25/17)

We are then to solve for y when x = 134

y = k√x

y = (25/17) × √134

y = (25/17) × 11.58

y = 17.02

Hope this Helps!!!

(x + y)² – 3zz when X = -2, y = -4, and z = 5.

Answers

-39

Explanation

[tex]\begin{gathered} \mleft(x+y\mright)^2-3zz \\ \end{gathered}[/tex]

Step 1

let

x=-2

y=-4

z=5

Step 2

Now, replace those values in the expression

[tex]\begin{gathered} (x+y)^2-3zz \\ (-2+(-4))^2-3\cdot5\cdot5 \\ (-2-4)^2-75 \\ (-6)^2-75 \\ 36-75 \\ -39 \end{gathered}[/tex]

I hope this helps you

if the endpoints of KB are K(-4, 5) and B(2, -5), what is the length of KB?

Answers

The length of the line can be found by distance formula as,

[tex]\begin{gathered} KB=\text{ }\sqrt[]{(x_2-x_1)^2+(y_2-y_1)^2} \\ KB=\sqrt[]{(2-(-4)_{})^2+(-5-5)^2} \\ KB=\sqrt[]{(2+4)^2+(-10)^2} \\ KB=\sqrt[]{6^2+100} \\ KB=\sqrt[]{36+100} \\ KB=\sqrt[]{136} \\ KB=11.66 \end{gathered}[/tex]

RATIOS, PROPORTIONS, AND PERCENTSFinding the principal, rate, or time for a simple interest loan...Ann took out a loan for $17,000 and was charged simple interest at an annual rate of 6.8%.The total interest she paid on the loan was $867.How long was the loan for, in months?Do not round any intermediate computations. If necessary, refer to the list of financial formulas.monthsI need help with this mathProblem

Answers

The time formula for simple interest is:

[tex]t=\frac{I}{Ci}[/tex]

Where I is the interest, C is the capital and i is the rate

In this case, we have:

I= 867

i=6.8%

C=17000

Replace and solve:

[tex]t=\frac{867}{17000*0.068}[/tex][tex]t=0.75\text{ years}[/tex]

The time is 0.75 years because the rate is in years.

Convert years to months:

[tex]0.75years*\frac{12months}{1years}=9\text{ months}[/tex]

I need help with number 7 the first question on the top of the page please

Answers

SK= 13x-5

KY= 2x+9

SY=36-x

By looking at the line segment we can state:

SY = SK+KY

Replacing with the values, and solving for x:

36-x=13x-5+2x+9

Sum and subtract alike terms

36+5-9=13x+2x+x

32= 16x

Divide both sides by 16:

32/16=16x/16

2=x

Replace the value of x in each expression:

SK= 13x-5=13(2)-5=26-5=21

KY= 2x+9=2(2)+9=4+9=13

SY=36-x =36-(2)=34

So, the answers are:

x=2

SK=21

KY=13

SY=34

A couple took a small airplane for a flight to the wine country for a romantic dinner and then returned home. The plane flew a total of 5 hours and each way the trip was 233 miles. If the plane was flying at 170 miles per hour, what was the speed of the wind that affected the plane?

Answers

Answer:

114.26 miles per hour

Explanation:

Let us call

v = wind speed

Then

speed with the wind = 170 + v

speed against the wind = 170 -v

Therefore,

The time taken on the outward journey ( with the wind):

[tex]\frac{233}{170+v}[/tex]

Time take on the return journey

[tex]\frac{233}{170-v}[/tex]

These two times must add up to 5 hours, the total time of the journey.

[tex]\frac{233}{170+v}+\frac{233}{170-v}=5[/tex]

Solving the above equation for v will give us the wind speed.

The first step is to find the common denominator of the two rational expressions. We do this by multiplying the left rational expression by (180-v)/(180-v) and the right expression by (180 + v)/(180 + v).

[tex]\frac{170-v}{170-v}*\frac{233}{170+v}+\frac{233}{170-v}*\frac{170+v}{170+v}=5[/tex][tex]\frac{233(170-v)+233(170+v)}{(170-v)(170+v)}=5[/tex]

Dividing both sides by 233 gives

[tex]\frac{(170-v)+(170+v)}{(170-v)(170+v)}=\frac{5}{233}[/tex]

The numerator on the left-hand side of the equation simplifies to give

[tex]\frac{2\times170}{(170-v)(170+v)}=\frac{5}{233}[/tex][tex]\Rightarrow\frac{340}{(170-v)(170+v)}=\frac{5}{233}[/tex]

Expanding the denominator gives

[tex]\operatorname{\Rightarrow}\frac{340}{170^2-v^2}=\frac{5}{233}[/tex][tex]\frac{340}{28900-v^2}=\frac{5}{233}[/tex]

Cross multipication gives

[tex]5(28900-v^2)=340\times233[/tex]

Dividing both sides by -5 gives

[tex]v^2-28900=-\frac{340\times233}{5}[/tex][tex]v^2-28900=-15844[/tex]

Adding 28900 to both sides gives

[tex]v^2=13056[/tex]

Finally, taking the sqaure root of both sides gives

[tex]\boxed{v=114.26.}[/tex]

Hence, the speed of the wind, rounded to two decimal places, was 114.26 miles per hour.

Need help with homework

Answers

Domain interval are the interval for the x - values on the linear graph

Therefore the domain intervals from the attached graph is

[tex]-3\leq x\leq4[/tex]

33<=105/p what is the answer

Answers

The answer is p≤35/11.

From the question, we have

33≤105/p

⇒p≤105/33

⇒p≤35/11

Inequality:

The mathematical expression with unequal sides is known as an inequality in mathematics. Inequality is referred to in mathematics when a relationship results in a non-equal comparison between two expressions or two numbers. In this instance, any of the inequality symbols, such as greater than symbol (>), less than symbol (), greater than or equal to symbol (), less than or equal to symbol (), or not equal to symbol (), is used in place of the equal sign "=" in the expression. Polynomial inequality, rational inequality, and absolute value inequality are the various types of inequalities that can exist in mathematics.

When the symbols ">", "", "", or "" are used to connect two real numbers or algebraic expressions, that relationship is known as an inequality.

To learn more about inequality visit: https://brainly.com/question/28823603

#SPJ9

PLS HELP FAST I WILL GIVE 25 POINTS simplify 10^6/10^-3. answers:A. 1/10^3. B. 1/10^18. C. 10^3. D. 10^9

Answers

D

Let's simplify that expression

1) Remember of the Exponents Rule, we have to subtract them

2) Note that for the exponents 6 -(-3) = 6+3 = 9 So it's D

Please help me solve question 6 on this algebra assignment

Answers

[tex]f(x)=\frac{3}{5}x-\frac{4}{3}[/tex]

At the zero of the function, f(x) = 0. Substituting f(x) = 0, we get:

[tex]0=\frac{3}{5}x-\frac{4}{3}[/tex]

Adding 4/3 at both sides of the equation:

[tex]\begin{gathered} 0+\frac{4}{3}=\frac{3}{5}x-\frac{4}{3}+\frac{4}{3} \\ \frac{4}{3}=\frac{3}{5}x \end{gathered}[/tex]

Multiplying by 5/3 at both sides of the equation:

[tex]\begin{gathered} \frac{5}{3}\cdot\frac{4}{3}=\frac{5}{3}\cdot\frac{3}{5}x \\ \frac{5\cdot4}{3\cdot3}=x \\ \frac{20}{9}=x \end{gathered}[/tex]

Therefore the coordinates of the zero of the function are:

[tex](x,f(x))=(\frac{20}{9},0)[/tex]

f(x)=4•2^2x,g(x)=2^4x+2, and h(x)=4^2x+1

Answers

Let's use the following property:

[tex]x^y\cdot x^z=x^{y+z}[/tex][tex]\begin{gathered} f(x)=4\cdot2^{2x} \\ g(x)=2^{4x+2}=2^{4x}\cdot2^2=4\cdot2^{4x} \\ h(x)=4^{2x+1}=4^{2x}\cdot2^{1^{}}=2\cdot4^{2x} \\ \text{Therefore:} \\ \text{None of them are equivalent} \end{gathered}[/tex]

Other Questions
For which machine did Grace Murray Hopper first write computer programs?O Analytical EngineMark IO UNIVACO ENIAC Given a horizontal uncharged insulating rod. A negatively charged rod touches the middle of the horizontal rod. Which of these best describes what happens? A)the rod becomes positively charged on its left end and negatively charged on its right end B)the middle of the rod becomes negatively charged C)the rod becomes positively charged all over its surface D)the rod becomes negatively charged all over its surfaceE) the rod remains uncharted Question 1(Multiple Choice Worth 2 points)(Converting Between Systems MC)I need help pleaseeeeFor a craft project you need 182 inches of ribbon, but it is only sold by the meter. Determine the amount of ribbon, in meters, you need to buy for the project. (1 inch = 2.54 centimeters and 1 centimeter = 0.01 meter) 462 47 12 5 Answer the question below. Are the angles congruent If yes, how do you know? Choose the line parallel to the following equation through the given point Pls help i need this answered quicklyTess builds a sandbox in the shape of a regular hexagon in which all the sides are 40 inches long. She then takes a photo of the sandbox and prints the photo. In the photo, the sides of the hexagon are each 4 inches long. Identify the scale factor that was used to create the printed photo of the sandbox. Express your answer as a decimal. A jet plane has a maximum deceleration rate of -7.30 m/s/s. It touches down on a runway with a speed of 122 m/s. From the instant it touches the runway, what is the minimum time needed before it comes to rest?? how do I solve x without measuring it, i need help with the third question please match the blanks to their missing phrases to complete the proof -2x - 14 =-2(Solve for x) A towns population is 53,350 about 100 people move out of the town each month 200 people on average move into town. A nearby town has a population of 56,375.It has no one moving in and an average of 175 people moving away every month in about how many months will be population of the town be EQUAL? This question is typical on some drivers license exams: A car moving at 46 km/h skids15 m with locked brakes.How far will the car skid with locked brakesat 115 km/h? Assume that energy loss is dueonly to sliding friction.Answer in units of m. What is the value of x?12 units15 units20 units25 units Write the first five terms of the geometric sequence with a1 = 16 and common ratio r= 3/2 These three pizzas are all the same size. Which one has the greatest number of equal pieces? What is mZADB in Circle D? 57 85.5 28.5 114 g a. three months after the sale, at the balance sheet date, there was a foreign currency exchange gain of $1100. what is the journal entry? b. three months after the sale, at the balance sheet date, there was a loss on the contract of $520. what is the journal entry? The location of a point moved from (1, - 3) to (-2, -1) by translation. Find the translation rule cuatro multiplicado por la suma de ocho y un numero.la suma de nueve y el numero